- Mon Sep 04, 2017 8:20 pm
#39213
Complete Question Explanation
(The complete setup for this game can be found here: lsat/viewtopic.php?t=12911)
The correct answer choice is (B)
If F and P are used together in a rug, we can safely rule out the 2-2-1 distribution, because it requires P and O to be used together in a rug. This deduction helps eliminate answer choice (A).
If F and P are two of the three colors in a rug subject to the 3-1-1 distribution, the third color must be W or O. This is because neither W nor O can be used in any of the solid rugs, as discussed in the main setup:
With no room for both W and O to be used in the multicolored rug, we need to ensure that one of them is not used in any of the rugs. The remaining two colors—T and Y—must be the colors of the two solid rugs:
By focusing our attention to the uncertainty inherent in this setup (W/O), we can easily see that answer choice (B) could be true, and is therefore correct.
Answer choice (A) is incorrect, because the applicable distribution here is 3-1-1, not 2-2-1, as discussed earlier.
Answer choice (B) is the correct answer choice, because W can be excluded without violating any of the conditions governing the assignment of colors to the three rugs.
Answer choice (C) is incorrect, because Y must be the color of one of the two solid rugs.
Answer choice (D) is incorrect, because T and W cannot be used together in any of the rugs.
Answer choice (E) is incorrect, because T and Y are the colors of the two solid rugs, and cannot be used together in any of the rugs.
(The complete setup for this game can be found here: lsat/viewtopic.php?t=12911)
The correct answer choice is (B)
If F and P are used together in a rug, we can safely rule out the 2-2-1 distribution, because it requires P and O to be used together in a rug. This deduction helps eliminate answer choice (A).
If F and P are two of the three colors in a rug subject to the 3-1-1 distribution, the third color must be W or O. This is because neither W nor O can be used in any of the solid rugs, as discussed in the main setup:
With no room for both W and O to be used in the multicolored rug, we need to ensure that one of them is not used in any of the rugs. The remaining two colors—T and Y—must be the colors of the two solid rugs:
By focusing our attention to the uncertainty inherent in this setup (W/O), we can easily see that answer choice (B) could be true, and is therefore correct.
Answer choice (A) is incorrect, because the applicable distribution here is 3-1-1, not 2-2-1, as discussed earlier.
Answer choice (B) is the correct answer choice, because W can be excluded without violating any of the conditions governing the assignment of colors to the three rugs.
Answer choice (C) is incorrect, because Y must be the color of one of the two solid rugs.
Answer choice (D) is incorrect, because T and W cannot be used together in any of the rugs.
Answer choice (E) is incorrect, because T and Y are the colors of the two solid rugs, and cannot be used together in any of the rugs.
You do not have the required permissions to view the files attached to this post.